LSAT problem example 2: what not to study

This topic has expert replies
User avatar
GMAT Instructor
Posts: 2193
Joined: Mon Feb 22, 2010 6:30 pm
Location: Vermont and Boston, MA
Thanked: 1186 times
Followed by:512 members
GMAT Score:770
Try the LSAT question below. This is a good example of the type of LSAT questions you should NOT use to study for the GMAT. Can you see why? Please offer thoughts as to why this is not like an official GMAT question.

And what about the answer - just for fun take a shot at getting it right -- I will explain later why to avoid this type of question so there is no pressure only the glory of getting it right!

"20. Ann will either take a leave of absence from Technocomp and return in a year or else she will quit her job there; but she would not do either one unless she were offered a one-year teaching fellowship at a prestigious university. Technocomp will allow her to take a leave of absence if it does not find out that she has been offered the fellowship but not otherwise. Therefore, Ann will quit her job at Technocomp only if Technocomp finds out she has been offered the fellowship.

Which of the following, if assumed, allows the conclusion above to be properly drawn?

A) Technocomp will find out about Ann being offered the fellowship only if someone informs on her.

B) The reason Ann wants the fellowship is so she can quit her job at Technocomp.

C) Technocomp does not allow any of its employees to take a leave of absence in order to work for one of its competitors.

D) Ann will take a leave of absence if Technocomp allows her to take a leave of absence.

E) Ann will be offered the fellowship only if she quit her job at Technocomp."

Source: Source: Official LSAT from December 1996, question 20 of section 2. Question found in "10 More Actual , Official PrepTests" copyright Law School Admissions Council, 2007.
Veritas Prep | GMAT Instructor

Veritas Prep Reviews
Save $100 off any live Veritas Prep GMAT Course

Master | Next Rank: 500 Posts
Posts: 437
Joined: Sat Nov 22, 2008 5:06 am
Location: India
Thanked: 50 times
Followed by:1 members
GMAT Score:580

by beat_gmat_09 » Fri Nov 19, 2010 6:32 am
One thing which i think is different - question stem, it uses "if assumed" in gmat i've seen "if true"
Correct me if i am wrong.
My pick is A.
If Technocomp finds out Ann has been offered fellowship, then Ann will quit her job.
For technocomp to find out someone needs to inform.
Hope is the dream of a man awake

User avatar
Master | Next Rank: 500 Posts
Posts: 154
Joined: Thu Aug 26, 2010 9:32 am
Location: Chicago,IL
Thanked: 46 times
Followed by:19 members
GMAT Score:760

by rkanthilal » Fri Nov 19, 2010 11:25 am
I agree with "beat_gmat_09" that the question stem is worded a little differently. That said, it seems equivalent to a GMAT assumption question... I can't spot any other differences.

I'll pick D for the answer.

User avatar
GMAT Instructor
Posts: 2193
Joined: Mon Feb 22, 2010 6:30 pm
Location: Vermont and Boston, MA
Thanked: 1186 times
Followed by:512 members
GMAT Score:770

by David@VeritasPrep » Fri Nov 19, 2010 12:51 pm
Nice answers guys...There is little more difference than that...is there any chance that this is not actually an assumption question?
Veritas Prep | GMAT Instructor

Veritas Prep Reviews
Save $100 off any live Veritas Prep GMAT Course

User avatar
Master | Next Rank: 500 Posts
Posts: 154
Joined: Thu Aug 26, 2010 9:32 am
Location: Chicago,IL
Thanked: 46 times
Followed by:19 members
GMAT Score:760

by rkanthilal » Fri Nov 19, 2010 2:50 pm
Wow... I still can't see it. This seems like an assumption question....

The conclusion is "Ann will quit her job at Technocomp only if Technocomp finds out she has been offered the fellowship."

The stated premises don't eliminate the possibility that Ann will quit her job even if Technocomp does not find out about the fellowship. My thinking was that in order to maintain this conclusion we need to assume that Ann will take the leave of absence if she can. Answer (D) pretty much says this.

I thought this was an assumption. I'm not sure...

User avatar
GMAT Instructor
Posts: 2193
Joined: Mon Feb 22, 2010 6:30 pm
Location: Vermont and Boston, MA
Thanked: 1186 times
Followed by:512 members
GMAT Score:770

by David@VeritasPrep » Fri Nov 19, 2010 4:25 pm
The above question might appear to be an assumption question - but it is not. On the GMAT there are really only two kinds of strengthen questions: regular strengthen questions and assumption questions. On the LSAT there is a third type of strengthen question where there answer choice is enough to guarantee the conclusion. There is no question type on the GMAT where the conclusion is guaranteed by the premise so this does not translate very well to the GMAT. Additionally, this type of question usually relies heavily on formal logic and is not a good choice to study.

How do you know what type of strengthen question an LSAT question is? Once you know that you have a strengthen question, there a two-step process to decide what type it is.

1) All regular strengthen questions on the LSAT use the word "most" as is "which of the following provides the most justification for the argument?" If you see the word most on a strengthen question use your strategy for attacking regular GMAT strengthen questions.

2) If the word most is not in the question stem the question will either be an assumption question, which may be good to study, or a question where the answer choice is sufficient to guarantee the conclusion, which is not good to study. Check to see if the question is as Assumption question by seeing if the argument makes an assumption, or if something is necessary or depended upon. Remember that this is the essence of an assumption question the argument depends on some answer choice. If there argument is not making an assumption and if it is not dependent on an answer choice then it is not an assumption and you should not study this problem.

Look at the question stem above...it says "Which of the following, if assumed, allows the conclusion above to be properly drawn?" You might ask why this is not an assumption question since it uses the word "assumed." It is true that anytime the word assumed is used on the GMAT it is an assumption question. But if you are going to use LSAT questions one more test is required. Try replacing the "if assumed" with "if true" as in "Which of the following, if true, allows the conclusion above to be properly drawn?" If you can replace the "assumed" with "true" then you can ignore it. And remember, as you learned from Sentence Correction, information set between commas is non-essential!

So we know that this is a type of question not to be found on the GMAT. Also, this question comes at number 20 on the test. As I indicated in the posting "Using the LSAT to study for the GMAT Part 2" questions 1 - 16 on LSAT critical reasoning sections are the best to study for the GMAT as later questions are often too complex. Finally, this question relies on formal logic, which is not required for the GMAT.

With that said, here is an explanation for the question above:

OA is D.

The conclusion is that Ann will quit her job only if Technocomp finds out that she has been offered the fellowship. Understand that this really means that if Technocomp does not found out that she has been offered the fellowship she will not quit her job. So, this conclusion is saying that if she either not offered the fellowship at all or if she is offered the fellowship but she can get a leave of absence then she will not quit her job. This makes sense right? No need to quit unless that is the only way to take the fellowship.

Now, we need to construct a series of premises that guarantees this to be true. The Premises we have now tell us that if Ann gets the fellowship she will take a leave of absence or quit her job. If she does not get the fellowship she will not do either of these (but just keep working). We are also told that if the company does not find out about the fellowship that they will offer her the leave of absence, but if they do find out she will not get the leave of absence. So what is the piece that is missing? Well the only hole in the argument is that Ann might be offered the leave of absence but not take it preferring to quit the job. (This is best illustrated using formal logic diagrams). Answer Choice D fills that gap by indicating that she would take the leave of absence if offered. Now the puzzle is complete. The conclusion is now guaranteed. The only way that she will quit is if she if offered the fellowship and the company finds out.
Veritas Prep | GMAT Instructor

Veritas Prep Reviews
Save $100 off any live Veritas Prep GMAT Course

User avatar
Master | Next Rank: 500 Posts
Posts: 154
Joined: Thu Aug 26, 2010 9:32 am
Location: Chicago,IL
Thanked: 46 times
Followed by:19 members
GMAT Score:760

by rkanthilal » Fri Nov 19, 2010 7:57 pm
That's interesting.... I did not know this about the LSAT.

David, thanks for this posting...

User avatar
Legendary Member
Posts: 1079
Joined: Mon Dec 13, 2010 1:44 am
Thanked: 118 times
Followed by:33 members
GMAT Score:710

by bblast » Mon Jun 06, 2011 7:56 am
@David u are the best person to answer this(As u know LSAT as well as GMAT inside out). Is the aristotle 101 cr from aristotle prep a a good study for GMAT ? they claim that the questions have been handpicked for gmat purposes.

Aristotle distributes this PDF as a free resource. All questions have been copied from valid public domain LSAT stuff.



For CR prep, This is the only LSAT set which I am planning to use besides OG12 and og verbal supplement.
Cheers !!

Quant 47-Striving for 50
Verbal 34-Striving for 40

My gmat journey :
https://www.beatthegmat.com/710-bblast-s ... 90735.html
My take on the GMAT RC :
https://www.beatthegmat.com/ways-to-bbla ... 90808.html
How to prepare before your MBA:
https://www.youtube.com/watch?v=upz46D7 ... TWBZF14TKW_